Answer (D) is correct . Because the call premium is $3, the stock price must be at least $63 ($60 exercise price + $3 call premium).
Answer (A) is incorrect because The amount of $57 is the result of deducting the call premium from the exercise price. Answer (B) is incorrect because The amount of $60 is the result of failing to consider the impact of the call premium. Answer (C) is incorrect because The full call premium must be added to the exercise price.
|